NURS 2500 Test 1 COMBINATION of all study guides in folder **Also must study Shayla's study guide

Ace your homework & exams now with Quizwiz!

After the nurse has finished teaching a 50-year-old female client about symptoms of coronary artery disease in women, which statement indicates that the teaching has been effective?

"I will call my health care provider about any unusual fatigue"

The nurse notes that a clients ABG results reveal: pH=7.50, PaCO2=30. The nurse monitors for which clinical manifestations associated with the results? SATA

-nausea -confusion -tachycardia -lightheadedness

The nurse notes that a clients ABG results reveal: pH=7.50, PaCO2=30. The nurse monitors the client for which clinical manifestations associated with these ABG results? SATA

-nausea -confusion -tachycardia -lightheadedness

The patient has type B COPD exacerbated by an acute upper respiratory infection. Which blood gas values should the nurse expect to see?

-pH low -PaCO2 is high -HCO3 is normal

The nurse is caring for a client with DKA and documents that the client is experiencing Kassmaul's respirations. Which patterns did the nurse observe? SATA

-respirations that are increased in rate -respiration that are abnormally deep

The nurse is caring for a client with a DKA and documents that the client is experiencing Kusmmaul's respirations. Which patterns did the nurse observe? SATA

-respirations that are increased in rate -respirations that are abnormally deep

The patient is hyperventilating from anxiety and abdominal pain. Which assessment findings should the nurse attribute to respiratory alkalosis? SATA

-tingling of the fingertips -numbness around mouth -cramping in feet

Glucose < 60 (70) newborn treatment =

0.5-1 g/kg of D12.5W via IV

Glucose < 60 (70) children treatment =

0.5-1 g/kg of D25W via IV

10. Which number corresponds to the spot where you would assess for an air leak in the patient with a chest tube?

1

The nurse is preparing to perform nasotracheal suctioning on a patient. Arrange the steps in order. 1. Perform hand hygiene. 2. Assist patient to semi-Fowler's or high Fowler's position, if able. 3. Apply sterile gloves. 4. Have patient take deep breaths. 5. Lubricate catheter with water-soluble lubricant. 6. Advance catheter through nares and into trachea. 7. Apply suction. 8. Withdraw catheter.

1. Perform hand hygiene. 2. Assist patient to semi-Fowler's or high Fowler's position, if able. 3. Apply sterile gloves. 4. Have patient take deep breaths. 5. Lubricate catheter with water-soluble lubricant. 6. Advance catheter through nares and into trachea. 7. Apply suction. 8. Withdraw catheter.

What is the most common preparation for insulin?

100 units per 1 mL

Normal glycemic control in 5-11 y

100-170

Normal glycemic control in < age 5yrs

110-200

What is the MAP? 168/92

117

Which diagnostic test is most important to obtain rapidly when caring for a client who has just arrived in the emergency department with possible acute coronary syndrome?

12-lead electrocardiogram (ECG)

A client has been admitted to the hospital for gastroenteritis and dehydration. The nurse determines that the client has received adequate volume replacement if the BUN level drops to which value?

15 mg/dL

Glucagon IM dosage for <8y = 0.5 mg while glucagon IM for >8y =

1mg

What are immediate actions needed to be taken with HHNS?

2 LARGE BORE IVs & IV replenishment with rapid 0.9 NS bolus of 30ml/kg infusion (labs STAT: CBC, CMP, serum osmo, lactic acid, amylase, CK, and blood cultures). Regular insulin IV. Prepare for insulin drip and 30 min to 1 hour BG testing. Cardiac monitoring and 12 Lead ECG. Prepare for possible infusions of potassium or high concentration sodium.

A client is admitted to the emergency department with crushing chest pain. A diagnosis of acute coronary syndrome is suspected. Which medication is indicated to prevent progression to a myocardial infarction?

Aspirin

Metoprolol is prescribed for a client with hypertension. The nurse monitors the client for which adverse effect?

Bradycardia

Nephropathy

stress on the nephrons filtering the glucose, starts leaking albumin

Cardiac output is determined by the stroke volume and what else?

stroke volume and heart rate

A patient who is taking potassium-wasting diuretic for treatment of hypertension reports generalized weakness. Which action is appropriate for the nurse to take?

suggest that the health care provider order a BMP

What drugs would you expect to be used for an active TB case?

•Isoniazid (hepatitis) •Rifampin (hepatitis; orange body fluids) •Pyrazinamide (hepatitis) •Ethambutol (ocular toxicity)

In the TMN classification system what does NX mean?

Nearby lymph nodes cannot be evaluated

vasodilator given for CP

Nitroglycerin

In the TMN classification system what does N0 mean?

No evidence of cancer cells in regional lymph nodes

In the TMN classification system what does M0 mean?

No evidence of metastases can be found

In the TMN classification system what does T0 mean?

No evidence of primary tumor (tumor cannot be found)

Is insulin dependency curable?

No, it's a lifelong dependency (insulin dependents cannot take metformin)

10-20 mg/dL

Normal BUN range

35-45

Normal CO2 range

22-26

Normal HCO3 range

7.35-7.45

Normal pH range

3.6-5.2 mmol/L

Normal potassium range

When teaching a group of clients about risk factors for heart disease, which topics will the nurse include? SATA

Obesity Hypertension

Which finding in a client seen in the emergency department with chest pain is most important to communicate to the health care provider?

ST segment elevation on electrocardiogram

Which finding for a client who has just returned to the nursing unit after an emergency cardiac catheterization would be most important to report to the primary health care provider?

ST-segment elevation on the electrocardiogram

a heart attack with ST-segment elevation

STEMI

Which information is most important to include when the nurse is teaching a client who has had an ST segment elevation myocardial infarction (STEMI) about the purpose of salt restriction?

Salt intake increases the work of the heart

What are the risk factors associated with lung cancer?

Smoking and exposure to secondhand smoke; Lung cancer can occur in non-smokers or people who weren't exposed to secondhand smoke, but it may never be clear the cause of the lung cancer; Exposure to radon gas (derived from uranium in the soil), asbestos, other carcinogens (arsenic, nickel, chromium); family history of lung cancer

Which information would the nurse include when teaching a client with coronary artery disease about aspirin therapy? SATA

Take aspirin with food Report ringing in the ears Monitor and report excessive bleeding and bruising Avoid over-the-counter pain medications that contain aspirin

When teaching about prevention of coronary artery disease (CAD) for a 50-year-old man who is 6 feet (183cm) tall and weighs 293 pounds (133kg), smokes 1 pack a day of cigarettes, and has siblings with CAD, which topics are most important for the nurse to include? SATA

Weight Tobacco use

Name at least 3 symptoms of Prostate cancer.

Weight loss, edema, fatigue, difficulty urinating or starting and maintaining stream, back pain, frequency especially at night, pain with urination and difficulty getting an erection

When is the best time to do genetic testing?

When a family history is present in at least 2 generations (go back to grandparents)

Which statement by the patient with type 2 diabetes is accurate? a. "I will limit my alcohol intake to 1 drink each day." b. "I am not allowed to eat any sweets because of my diabetes." c. "I cannot exercise because I take a blood glucose-lowering medication." d. "The amount of fat in my diet is not important. Only carbohydrates raise my blood sugar."

a. "I will limit my alcohol intake to 1 drink each day."

Analyze the following diagnostic findings for your patient with type 2 diabetes. Which result will need further assessment? a. A1C 9% b. BP 126/80 mmHg c. FBG 130 mg/dL (7.2 mmol/L) d. LDL cholesterol 100 mg/dL (2.6 mmol/L)

a. A1C 9%

A patient has a severe blockage in his right coronary artery. Which heart structures are most likely to be affected by this blockage (select all that apply)? a. AV node b. Left ventricle c. Coronary sinus d. Right ventricle e. Pulmonic valve

a. AV node b. Left ventricle d. Right ventricle

What features of cancer cells distinguish them from normal cells (select all that apply)? a. Cells lack contact inhibition. b. Cells undergo rapid proliferation. c. Cells return to a previous undifferentiated state. d. Proliferation occurs when there is a need for more cells. e. New proteins characteristic of embryonic stage emerge on cell membrane.

a. Cells lack contact inhibition. c. Cells return to a previous undifferentiated state. e. New proteins characteristic of embryonic stage emerge on cell membrane.

A patient with newly discovered high blood pressure has an average reading of 158/98 mm Hg after 3 months of exercise and diet modifications. Which management strategy will e a priority for this patient? a. Drug therapy will be needed because the bp is still not at goal b. Bp monitoring should continue for 3 months to confirm a diagnosis of hypertension c. Lifestyle changes are less important since they were not effective, and drugs will be started d. More changes in the patient's lifestyle are needed for a longer time before starting drug therapy

a. Drug therapy will be needed because the bp is still not at goal

A 35-yr-old female patient is admitted to the emergency department with acute abdominal pain. Which medical diagnoses should you consider as possible causes of her pain? (select all that apply) a. Gastroenteritis b. Ectopic pregnancy c. Gastrointestinal bleeding d. Irritable bowel syndrome e. Inflammatory bowel disease

a. Gastroenteritis b. Ectopic pregnancy c. Gastrointestinal bleeding d. Irritable bowel syndrome e. Inflammatory bowel disease

A 70-year-old male patient has multiple myeloma. His wife calls to report that he sleeps most of the day, is confused when awake, and reports nausea and constipation. Which complication of cancer is this most likely caused by? a. Hypercalcemia b. Tumor lysis syndrome c. Spinal cord compression d. Superior vena cava syndrome

a. Hypercalcemia

You are caring for a patient with newly diagnosed type 1 diabetes. What information is essential to include in your patient teaching before discharge from the hospital? (select all that apply) a. Insulin administration b. Elimination of sugar from diet c. Need to reduce physical activity d. Use of a portable blood glucose monitor e. Hypoglycemia prevention, symptoms, and treatment

a. Insulin administration d. Use of a portable blood glucose monitor e. Hypoglycemia prevention, symptoms, and treatment

A patient has recently been diagnosed with early stages of breast cancer. What is most appropriate for the nurse to focus on? a. Maintaining the patient's hope b. Preparing a will and advance directives c. Discussing replacement child care for the patient's children d. Discussing the patient's past experiences with her grandmother's cancer

a. Maintaining the patient's hope

A patient is admitted to the hospital in a hypertensive emergency (bp 244/142). Sodium nitroprusside is started to treat he elevated bp. Which management strategies would be most appropriate for the patient? (select all that apply) a. Measuring hourly urine output b. Continuous bp monitoring with an arterial line c. Decreasing the MAP by 50% within the first hour d. Maintaining bed rest and giving tranquilizers to lower the bp e. Assessing the patient for signs and symptoms of heart failure and changes in mental status

a. Measuring hourly urine output b. Continuous bp monitoring with an arterial line e. Assessing the patient for signs and symptoms of heart failure and changes in mental status

Which nursing responsibilities are priorities when caring for a patient returning from a cardiac catheterization (select all that apply)? a. Monitoring vital signs and ECG b. Checking the catheter insertion site and distal pulses c. Assisting the patient to ambulate to the bathroom to void d. Informing the patient that he will be sleepy from the general anesthesia e. Instructing the patient about the risks of the radioactive isotope injection

a. Monitoring vital signs and ECG b. Checking the catheter insertion site and distal pulses

The nurse is caring for a patient with a chest tube for treatment of a right pneumothorax. Which assessment finding necessitates immediate notification of the health care provider? a. New, vigorous bubbling in the water seal chamber. b. Scant amount of sanguineous drainage noted on the dressing. c. Clear but slightly diminished breath sounds on the right side of the chest. d. Pain score of 2 one hour after the administration of the prescribed analgesic.

a. New, vigorous bubbling in the water seal chamber.

Which factors would place a patient at a higher risk for prostate cancer (select all that apply)? a. Older than 65 years b. Asian or Native American c. Long-term use of an indwelling urethral catheter d. Father diagnosed and treated for early-stage prostate cancer e. Previous history of undescended testicle and testicular cancer

a. Older than 65 years d. Father diagnosed and treated for early-stage prostate cancer

Which patients have the greatest risk for aspiration pneumonia? (select all that apply) a. Patient with seizures b. Patient with head injury c. Patient who had thoracic surgery d. Patient who had a myocardial infarction e. Patient who is receiving nasogastric tube feeding

a. Patient with seizures b. Patient with head injury e. Patient who is receiving nasogastric tube feeding

8. The nurse has just witnessed her patient go into cardiac arrest. What priority interventions should the nurse perform at this time? (Select all that apply.) a. Perform chest compressions. b. Ask someone to bring the defibrillator to the room for immediate defibrillation. c. Apply oxygen via nasal cannula. d. Place the patient in the high Fowler's position. e. Educate the family about the need for CPR.

a. Perform chest compressions. b. Ask someone to bring the defibrillator to the room for immediate defibrillation.

A nurse plans a community education program related to prevention of the cancer with the highest death rates in both women and men. What should the nurse include in the teaching plan? a. Smoking cessation; b. Screening with colonoscopy; c. Regular examination of reproductive organs; d. Use of sunscreen as protection from ultraviolet light

a. Smoking cessation

In assessing patients for STIs, the nurse needs to know that many STIs can be asymptomatic. Which STIs can be asymptomatic? (select all that apply) a. Syphilis b. Gonorrhea c. Genital warts d. Genital herpes e. Chlamydial infection

a. Syphilis b. Gonorrhea c. Genital warts d. Genital herpes e. Chlamydial infection

The nurse is caring for a patient with an artificial airway. What are reasons to suction the patient? (Select all that apply.) a. The patient has visible secretions in the airway. b. There is a sawtooth pattern on the patient's EtCO2 monitor. c. The patient has clear breath sounds. d. It has been 3 hours since the patient was last suctioned. e. The patient has excessive coughing.

a. The patient has visible secretions in the airway. b. There is a sawtooth pattern on the patient's EtCO2 monitor. e. The patient has excessive coughing.

The nurse is preparing the patient for a diagnostic procedure to remove pleural fluid for analysis. The nurse would prepare the patient for which test? a. Thoracentesis b. Bronchoscopy c. Pulmonary angiography d. Sputum culture and sensitivity

a. Thoracentesis

The nurse determines that a patient with which disorder is most at risk for spreading the disease? a. Tinea pedis b. Impetigo on the face c. Candidiasis of the nails d. Psoriasis on the palms and soles

a. Tinea pedis

Which populations have a higher risk for acquiring sexually transmitted infections (STIs)? (select all that apply) a. Transgender persons b. Young adults (age < 25) c. Men who have sex with men d. Men in long-term care facilities e. Women in correctional facilities

a. Transgender persons b. Young adults (age < 25) c. Men who have sex with men e. Women in correctional facilities

Which are appropriate therapies for patients with diabetes? (select all that apply) a. Use of statins to reduce CVD risk b. Use of diuretics to treat nephropathy c. Use of ACE inhibitors to treat nephropathy d. Use of serotonin agonists to decrease appetite e. Use of laser photocoagulation to treat retinopathy

a. Use of statins to reduce CVD risk c. Use of ACE inhibitors to treat nephropathy e. Use of laser photocoagulation to treat retinopathy

Which safe sun practices would the nurse include in the teaching plan for a patient who has photosensitivity (select all that apply)? a. Wear protective clothing. b. Apply sunscreen liberally and often. c. Emphasize the short-term use of a tanning booth. d. Avoid exposure to the sun, especially during midday. e. Wear any sunscreen as long as it is bought at a drugstore.

a. Wear protective clothing. b. Apply sunscreen liberally and often. d. Avoid exposure to the sun, especially during midday.

A student nurse asks the RN what can be measured by arterial blood gas (ABG). The RN tells the student that the ABG can measure (select all that apply) a. acid-base balance b. oxygenation status c. acidity of the blood d. bicarbonate (HCO3-) e. compliance and resistance

a. acid-base balance b. oxygenation status c. acidity of the blood d. bicarbonate (HCO3-)

A mother and her two children have been diagnosed with pediculosis corporis at a health care center. An appropriate measure to treat this condition is a. applying pyrethrins to the body. b. topical application of griseofulvin. c. moist compresses applied frequently. d. administration of systemic antibiotics.

a. applying pyrethrins to the body.

The nurse is obtaining a subjective data assessment from a woman reported as a sexual contact of a man with chlamydial infection. The nurse understands that symptoms of chlamydial infection in women a. are often absent. b. are similar to those of genital herpes. c. include a macular palmar rash in the later stages. d. may involve chancres inside the vagina that are not visible.

a. are often absent.

When caring for a patient with acute bronchitis, the nurse will prioritize interventions by: a. auscultating lung sounds b. encouraging fluid restriction c. administering antibiotic therapy d. teaching the patient to avoid cough suppressants

a. auscultating lung sounds

Important patient teaching after a chemical peel includes a. avoidance of sun exposure. b. application of firm bandages. c. limitation of vigorous exercise. d. use of moist heat to relieve discomfort.

a. avoidance of sun exposure.

When a person's blood pressure rises, the homeostatic mechanism to compensate for an elevation involves stimulation of a. baroreceptors that inhibit the sympathetic nervous system, causing vasodilation b. chemoreceptors that inhibit the sympathetic nervous system causing vasodilation c. baroreceptors that inhibit the parasympathetic nervous system, causing vasodilation d. chemoreceptors that stimulate the sympathetic nervous system, causing an increased heart rate

a. baroreceptors that inhibit the sympathetic nervous system, causing vasodilation

A patient has been receiving high-dose corticosteroids and broad-spectrum antibiotics for treatment of an infection after a traumatic injury. The nurse plans care for the patient knowing that the patient is most susceptible to a. candidiasis b. cryptococcosis c. histoplasmosis d. coccidioidomycosis

a. candidiasis

The key anatomic landmark that separates the upper respiratory tract from the lower respiratory tract is the a. carina b. larynx c. trachea d. epiglottis

a. carina

It is important for the nurse to assess for which manifestation(s) in a patient who has just undergone a total thyroidectomy? (select all that apply) a. confusion b. weight gain c. depressed reflexes d. circumoral numbness e. positive Chvostek's sign

a. confusion d. circumoral numbness e. positive Chvostek's sign

The nurse notes tidaling of the water level in the water-seal chamber in a patient with closed chest tube drainage. The nurse should a. continue to monitor the patient b. check all connections for a leak in the system c. lower the drainage collector further from the chest d. clamp the tubing at the distal point away from the patient

a. continue to monitor the patient

Defense mechanisms that help protect the lung from inhaled particles and microorganisms include the (select all that apply). a. cough reflex b. mucociliary escalator c. alveolar macrophages d. reflex bronchoconstriction e. alveolar capillary membrane

a. cough reflex b. mucociliary escalator c. alveolar macrophages d. reflex bronchoconstriction

When caring for a patient with thrombocytopenia, the nurse instructs the patient to a. dab his or her nose instead of blowing. b. be careful when shaving with a safety razor. c. continue with physical activities to stimulate thrombopoiesis. d. avoid aspirin because it may mask the fever that occurs with thrombocytopenia.

a. dab his or her nose instead of blowing.

When assessing a patient, you note a pulse deficit of 23 beats. This finding may be caused by a. dysrhythmias b. heart murmurs c. gallop rhythms d. pericardial friction rubs

a. dysrhythmias

The nursing care for a patient with hyponatremia and fluid volume excess includes a. fluid restriction b. administration of hypotonic IV fluids c. administration of a cation-exchange resin d. placement of an indwelling urinary catheter

a. fluid restriction

A priority nursing intervention for a patient who has just undergone a chemical pleurodesis for recurrent pleural effusion is a. giving ordered analgesia b. monitoring chest tube drainage c. sending pleural fluid for laboratory analysis d. monitoring the patient's level of consciousness

a. giving ordered analgesia

The most appropriate therapy for a patient with acute diarrhea caused by a viral infection is to a. increase fluid intake. b. administer an antibiotic. c. administer an antimotility drug. d. quarantine the patient to prevent spread of the virus.

a. increase fluid intake.

The lungs act as an acid-base buffer by: a. increasing respiratory rate and depth when CO2 levels in the blood are high, reducing acid load. b. increasing respiratory rate and depth when CO2 levels in the blood are low, reducing base load c. decreasing respiratory rate and depth when CO2 levels in the blood are high, reducing acid load. d. decreasing respiratory rate and depth when CO2 levels in the blood are low, increasing acid load.

a. increasing respiratory rate and depth when CO2 levels in the blood are high, reducing acid load.

The nursing management of a patient in sickle cell crisis includes (select all that apply) a. monitoring CBC. b. optimal pain management and O2 therapy. c. blood transfusions if needed and iron chelation. d. rest as needed and deep vein thrombosis prophylaxis. e. administration of IV iron and diet high in iron content.

a. monitoring CBC. b. optimal pain management and O2 therapy. c. blood transfusions if needed and iron chelation. d. rest as needed and deep vein thrombosis prophylaxis.

A patient is admitted to the ICU with a diagnosis of NSTEMI. Which drug(s) would the nurse expect the patient to receive? (select all that apply) a. oral statin therapy b. antiplatelet therapy c. thrombolytic therapy d. prophylactic antibiotics e. intravenous nitroglycerin

a. oral statin therapy b. antiplatelet therapy e. intravenous nitroglycerin

The nurse performs a detailed assessment of the abdomen of a patient with a possible bowel obstruction, knowing that manifestations of an obstruction in the large intestine are (select all that apply) a. persistent abdominal pain. b. marked abdominal distention. c. diarrhea that is loose or liquid. d. colicky, severe, intermittent pain. e. profuse vomiting that relieves abdominal pain.

a. persistent abdominal pain. b. marked abdominal distention.

You are caring for patients exposed to a chlorine leak from a local factory. The nurse would closely monitor these patients for a. pulmonary edema b. anaphylactic shock c. respiratory alkalosis d. acute tubular necrosis

a. pulmonary edema

Assessment findings suggestive of peritonitis include (select all that apply) a. rebound tenderness. b. a soft, distended abdomen. c. dull, intermittent abdominal pain. d. shallow respirations with bradypnea. e. observing that the patient is lying still.

a. rebound tenderness. e. observing that the patient is lying still.

Which BP regulating mechanism(s) can result in the development of hypertension if defective (select all that apply)? a. release of norepinephrine b. secretion of prostaglandins c. stimulation of the sympathetic nervous system d. stimulation of the parasympathetic nervous system e. activation of the renin-angiotensin-aldosterone system

a. release of norepinephrine c. stimulation of the sympathetic nervous system e. activation of the renin-angiotensin-aldosterone system

The nurse determines that the goals of dietary teaching have been met when the patient with celiac disease selects from the menu a. scrambled eggs and sausage. b. buckwheat pancakes with syrup. c. oatmeal, skim milk, and orange juice. d. yogurt, strawberries, and rye toast with butter.

a. scrambled eggs and sausage.

Priority nursing actions when caring for a hospitalized patient with a new-onset temperature of 102.2°F (39°C) and severe neutropenia include (select all that apply) a. starting the prescribed antibiotic STAT. b. drawing peripheral and central line blood cultures. c. ongoing monitoring of the patient's vital signs for septic shock. d. taking a full set of vital signs and notifying the physician immediately. e. administering transfusions of WBCs treated to decrease immunogenicity.

a. starting the prescribed antibiotic STAT. b. drawing peripheral and central line blood cultures. c. ongoing monitoring of the patient's vital signs for septic shock. d. taking a full set of vital signs and notifying the physician immediately.

The primary protective role of the immune system related to malignant cells is a. surveillance for cells with tumor-associated antigens. b. binding with free antigens released by all cancer cells. c. producing blocking factors that immobilize cancer cells. d. reacting to a new set of antigenic determinants on cancer cells.

a. surveillance for cells with tumor-associated antigens.

When teaching a patient with melanoma about this disorder, the nurse recognizes that the patient's prognosis is most dependent on a. the thickness of the lesion. b. the degree of asymmetry in the lesion. c. the amount of ulceration in the lesion. d. how much the lesion has spread superficially.

a. the thickness of the lesion.

A complication of the hyperviscosity of polycythemia is a. thrombosis. b. cardiomyopathy. c. pulmonary edema. d. disseminated intravascular coagulation (DIC).

a. thrombosis.

An older woman is admitted to the medical unit with GI bleeding. Assessment findings that indicated fluid volume deficit include (select all that apply) a. weight loss b. dry oral mucosa c. full bounding pulse d. engorged neck veins e. decreased central venous pressure

a. weight loss b. dry oral mucosa e. decreased central venous pressure

RX known to cause angioedema

ace inhibitor

The nurse observes that a patient with respiratory disease experiences a decrease in SpO2 from 93% to 88% while ambulating. What is the priority action of the nurse?

administer PRN supplemental O2.

A patient with DKA has rapid, deep respirations. Which action should the nurse take?

administer the prescribed fluid bolus and insulin

How do chemotherapy drugs work? Chemotherapy drugs

affect the S phase of cell reproduction by interfering with DNA and RNA synthesis

A patient with new-onset confusion and hyponatremia is being admitted. Which action should the charge nurse take when making room assignments?

assign the patient to a room near the nursing station

Type 2 diabetes in adults...insulin resistance occurs with age, obesity, high fat high calorie diet, sedentary lifestyle, family history but is also

associated with low HDL cholesterol, high triglycerides, high blood pressure, truncal obesity as well as chronic steroid use, schizophrenia, depression treated with 1st and 2nd generation antidepressants, history of gestational diabetes, PCOS, C-V diagnosis, MI, and CVA

When is a colonoscopy first recommended

at the age of 50 or earlier if family history of colorectal cancer

After placement of a centrally inserted IV catheter, a patient reports acute chest pain and dyspnea. Which action should the nurse take first?

auscultate the patients breath sounds

Skin cancer - Squamous cell carcinoma prevention includes:

avoid sun during middle of day; wear sunscreen year round; wear protective clothing; avoid tanning beds; check skin regularly and report changes to doctor

After teaching about ways to decrease risk factors for CAD, the nurse recognizes that further instruction is needed when the patient says a. "I can keep my bp normal with medication." b. "I would like to add weight lifting to my exercise program." c. "I can change my diet to decrease my intake of saturated fats." d. "I will change my lifestyle to reduce activities that increase my stress."

b. "I would like to add weight lifting to my exercise program."

The nurse is performing discharge teaching for a patient with chronic obstructive pulmonary disease (COPD). What statement, made by the patient, indicates the need for further teaching? a. "Pursed-lip breathing is like exercise for my lungs and will help me strengthen my breathing muscles." b. "When I am sick, I should limit the amount of fluids I drink so that I don't produce excess mucus." c. "I will ensure that I receive an influenza vaccine every year, preferably in the fall." d. "I will look for a smoking-cessation support group in my neighborhood."

b. "When I am sick, I should limit the amount of fluids I drink so that I don't produce excess mucus."

The nurse should explain to the patient who has erectile dysfunction (ED) that (select all that apply) a. the most common cause is benign prostatic hypertrophy. b. ED may be due to medications or conditions such as diabetes. c. only men who are 65 years or older benefit from PDE5 inhibitors. d. there are medications and devices that can be used to help with erections. e. this condition is primarily due to anxiety and best treated with psychotherapy.

b. ED may be due to medications or conditions such as diabetes. d. there are medications and devices that can be used to help with erections.

The nurse is caring for a patient with pneumonia. On entering the room, the nurse finds the patient lying in bed, coughing, and unable to clear secretions. What should the nurse do first? a. Start oxygen at 2 L/min via nasal cannula. b. Elevate the head of the bed to 45 degrees. c. Encourage the patient to use the incentive spirometer. d. Notify the health care provider.

b. Elevate the head of the bed to 45 degrees.

Which heart valve sound is heard best at the left midclavicular line at the level of the fifth ICS? a. Aortic b. Mitral c. Tricuspid d. Pulmonic

b. Mitral

9. The nurse is performing tracheostomy care on a patient. What finding would indicate that the tracheostomy tube has become dislodged? a. Clear breath sounds b. Patient speaking to nurse c. SpO2 reading of 96% d. Respiratory rate of 18 breaths/minute

b. Patient speaking to nurse

Which assessment findings indicate that the patient is experiencing an acute disturbance in oxygenation and requires immediate intervention? (Select all that apply.) a. SpO2 value of 95% b. Retractions c. Respiratory rate of 28 breaths per minute d. Nasal flaring e. Clubbing of fingers

b. Retractions c. Respiratory rate of 28 breaths per minute d. Nasal flaring

A P wave on an ECG represents an impulse arising at the a. SA node and repolarizing the atria. b. SA node and depolarizing the atria. c. AV node and depolarizing the atria. d. AV node and spreading to the bundle of His.

b. SA node and depolarizing the atria.

Which subjective data related to the cardiovascular system should be obtained from the patient (select all that apply)? a. Annual income b. Smoking history c. Religious preference d. Number of pillows used to sleep e. Blood for basic laboratory studies

b. Smoking history c. Religious preference d. Number of pillows used to sleep

Which statement(s) describe(s) the management of a patient following lung transplantation (select all that apply) a. High dose of O2 are administered around the clock b. Using a home spirometer will help to monitor lung function c. Immunosuppressant therapy usually involves a 3-drug regimen d. Most patients have an acute rejection episode within the first 2 days e. A lung biopsy is done using a transtracheal method if rejection is suspected

b. Using a home spirometer will help to monitor lung function c. Immunosuppressant therapy usually involves a 3-drug regimen e. A lung biopsy is done using a transtracheal method if rejection is suspected

In teaching a patient about coronary artery disease, the nurse explains that the changes that occur in this disorder include (select all that apply) a. diffuse involvement of plaque formation in coronary veins b. abnormal levels of cholesterol, especially low-density lipoproteins c. accumulation of lipid and fibrous tissue within the coronary arteries d. development of angina due to a decreased blood supply to the heart muscle e. chronic vasoconstriction of coronary arteries leading to permanent vasospasm

b. abnormal levels of cholesterol, especially low-density lipoproteins

To prevent fever and shivering during an infusion of rituximab (Rituxan), the nurse should premedicate the patient with a. aspirin. b. acetaminophen. c. sodium bicarbonate. d. meperidine (Demerol).

b. acetaminophen.

To detect early signs or symptoms of inadequate oxygenation, the nurse would examine the patient for a. dyspnea and hypotension b. apprehension and restlessness c. cyanosis and cool, clammy skin d. increased urine output and diaphoresis

b. apprehension and restlessness

When auscultating the chest of an older patient in mild respiratory distress, it is best to a. begin listening at the apices b. begin listening at the lung bases c. begin listening on the anterior chest d. ask the patient to breathe through the nose with the mouth closed

b. begin listening at the lung bases

The nurse expects the long-term treatment of a patient with hyperphosphatemia from renal failure will include a. fluid restriction b. calcium supplements c. magnesium supplements d. increased intake of dairy products

b. calcium supplements

A hospitalized patient with a history of chronic stable angina tells the nurse that she is having chest pain. The nurse bases his actions on the knowledge that ischemia a. will always progress to myocardial infarction b. can be relieved by rest, nitroglycerin, or both c. is often associated with vomiting and extreme fatigue d. indicates that irreversible myocardial damage is occurring

b. can be relieved by rest, nitroglycerin, or both

When obtaining assessment data from a patient with a microcytic, hypochromic anemia, the nurse would ask the patient about a. folic acid intake. b. dietary intake of iron. c. a history of gastric surgery. d. a history of sickle cell anemia.

b. dietary intake of iron.

When assessing subjective data related to the respiratory health of a patient with emphysema, the nurse asks about (select all that apply) a. date of last chest x-ray b. dyspnea during rest or exercise c. pulmonary function test results d. ability to sleep through the entire night e. prescription and over-the-counter medication

b. dyspnea during rest or exercise d. ability to sleep through the entire night e. prescription and over-the-counter medication

The part of the vascular system responsible for hemostasis is the a. thin capillary vessels b. endothelial layer of the arteries c. elastic middle layer of the veins d. smooth muscle of the arterial wall

b. endothelial layer of the arteries

A patient scheduled for a radical prostatectomy for prostate cancer expresses the fear that he will have erectile dysfunction. In responding to this patient, the nurse should keep in mind that a. PD5 inhibitors are not recommended in prostatectomy patients. b. erectile dysfunction can occur even with a nerve-sparing procedure. c. the most common complication of this surgery is bowel incontinence. d. the provider will place a penile implant during surgery to treat any dysfunction.

b. erectile dysfunction can occur even with a nerve-sparing procedure.

While obtaining subjective assessment data from a patient with hypertension, the nurse recognizes that a modifiable risk factor for the development of hypertension is a. a low-calcium diet b. excessive alcohol consumption c. a family history of hypertension d. consumption of a high-protein diet

b. excessive alcohol consumption

Polydipsia and polyuria related to diabetes are primarily due to a. the release of ketones from cells during fat metabolism. b. fluid shifts resulting from the osmotic effect of hyperglycemia. c. damage to the kidneys from exposure to high levels of glucose. d. changes in RBCs resulting from attachment of excess glucose to hemoglobin.

b. fluid shifts resulting from the osmotic effect of hyperglycemia.

The nurse receives an order for a patient with lung cancer to receive influenza vaccine and pneumococcal vaccines. The nurse will a. call the health care provider to question the order b. give both vaccines at the same time in different arms c. give the pneumococcal vaccine and obtain a nasal influenza vaccine d. give the flu shot and tell the patient to come back in a week to have the pneumococcal vaccine

b. give both vaccines at the same time in different arms

When reviewing a patient's hematologic laboratory values after a splenectomy, the nurse would expect to find a. RBC abnormalities. b. increased WBC count. c. decreased hemoglobin. d. decreased platelet count.

b. increased WBC count.

The nurse is caring for a patient who is 2 days post MI. The patient reports that she is experiencing chest pain when she takes a deep breath. Which action would be a priority? a. notify the provider STAT and obtain a 12-lead ECG b. obtain vital signs and auscultate for a pericardial friction rub c. apply high-flow O2 by face mask and auscultate breath sounds d. medicate the patient with as-needed analgesic and reevaluate in 30 minutes

b. obtain vital signs and auscultate for a pericardial friction rub

In contrast to diverticulitis, the patient with diverticulosis a. has rectal bleeding. b. often has no symptoms. c. usually develops peritonitis. d. has localized cramping pain.

b. often has no symptoms.

The nurse explains to the patient with chronic bacterial prostatitis who is undergoing antibiotic therapy that (select all that apply) a. all patients require hospitalization. b. pain will lessen once treatment has ended. c. the course of treatment is generally 1 to 2 weeks. d. long-term therapy may be needed in immunocompromised patient. e. if the condition is not treated appropriately, he is at risk for prostate cancer.

b. pain will lessen once treatment has ended. d. long-term therapy may be needed in immunocompromised patient.

In assessing a patient for testicular cancer, the nurse understands that the manifestations of this disease often include a. urinary frequency. b. painless mass in the scrotal area. c. erectile dysfunction with retrograde ejaculation. d. rapid onset of dysuria with scrotal swelling and fever.

b. painless mass in the scrotal area.

During the assessment of a patient, you note an area of red, sharply defined plaques covered with silvery scales that are mildly itchy on the patient's knees and elbows. You would describe this finding as a. lentigo. b. psoriasis. c. actinic keratosis. d. seborrheic keratosis.

b. psoriasis.

The nurse explains to a patient undergoing brachytherapy of the cervix that she a. must undergo simulation to locate the treatment area. b. requires the use of radioactive precautions during nursing care. c. may have desquamation of the skin on the abdomen and upper legs. d. requires shielding of the ovaries during treatment to prevent ovarian damage.

b. requires the use of radioactive precautions during nursing care.

In teaching a patient who is using topical corticosteroids to treat acute dermatitis, the nurse should tell the patient that (select all that apply) a. the cream form is the most efficient system of delivery. b. short-term use of topical corticosteroids usually does not cause systemic side effects. c. use a glove to apply small amounts of creams or ointments to prevent further infection. d. abruptly stopping the use of topical corticosteroids may cause the dermatitis to reappear. e. systemic side effects from topical corticosteroids are likely if the patient is malnourished.

b. short-term use of topical corticosteroids usually does not cause systemic side effects. d. abruptly stopping the use of topical corticosteroids may cause the dermatitis to reappear.

Provide emotional support to a patient with an STI by a. offering information on how safer sexual practices can prevent STIs. b. showing concern when listening to the patient who expresses negative feelings. c. reassuring the patient that the disease is highly curable with appropriate treatment. d. helping the patient who received an STI from their sexual partner in forgiving the partner.

b. showing concern when listening to the patient who expresses negative feelings.

A primary HSV infection differs from recurrent HSV episodes in that (select all that apply) a. only primary infections are sexually transmitted. b. symptoms are less severe during recurrent episodes. c. transmission of the virus to a fetus is less likely during primary infection. d. systemic manifestations, such as fever and myalgia, are more common in primary infection. e. lesions from recurrent HSV are more likely to transmit the virus than lesions from primary HSV.

b. symptoms are less severe during recurrent episodes. d. systemic manifestations, such as fever and myalgia, are more common in primary infection.

When planning care for a patient at risk for pulmonary embolism, the nurse prioritizes a. maintaining the patient on bed rest b. using intermittent pneumatic compression devices c. encouraging the patient to cough and deep breath d. teaching the patient how to use the incentive spirometer

b. using intermittent pneumatic compression devices

reduces BP, HR, and contractility

beta blocker

Which patient is at greatest risk for developing hypermagnesemia? a. 83 year old man with lung cancer and hypertension b. 65 year old woman with hypertension taking B-adrenergic blockers c. 42 year old woman with systemic lupus erythematosus and renal failure d. 50 year old man with benign prostatic hyperplasia and a urinary tract infection

c. 42 year old woman with systemic lupus erythematosus and renal failure

A patient with TB has been admitted to the hospital and is placed on airborne precautions and in an isolation room. What should the nurse teach the patient? (select all that apply) a. Expect routine TB testing to evaluate the infection b. No visitors will be allowed while in airborne isolation c. Adherence to precautions includes coughing into a paper tissue. d. Take all medications for full length of time to prevent multi-drug-resistant TB e. Wear a standard isolation mask if leaving the airborne infection isolation room

c. Adherence to precautions includes coughing into a paper tissue. d. Take all medications for full length of time to prevent multi-drug-resistant TB e. Wear a standard isolation mask if leaving the airborne infection isolation room

A patient on chemotherapy and radiation for head and neck cancer has a WBC count of 1.9 × 103/μL, hemoglobin of 10.8 g/dL, and a platelet count of 99 × 103/μL. Based on the CBC results, what is the most serious clinical finding? a. Cough, rhinitis, and sore throat b. Fatigue, nausea, and skin redness at site of radiation c. Temperature of 101.9° F, fatigue, and shortness of breath d. Skin redness at site of radiation, headache, and constipation

c. Temperature of 101.9° F, fatigue, and shortness of breath

The goals of cancer treatment are based on the principle that a. surgery is the single most effective treatment for cancer. b. initial treatment is always directed toward cure of the cancer. c. a combination of treatment modalities is effective for controlling many cancers. d. although cancer cure is rare, quality of life can be increased with treatment modalities.

c. a combination of treatment modalities is effective for controlling many cancers.

DIC is a disorder in which a. the coagulation pathway is genetically altered, leading to thrombus formation in all major blood vessels. b. an underlying disease depletes hemolytic factors in the blood, leading to diffuse thrombotic episodes and infarcts. c. a disease process stimulates coagulation processes with resultant thrombosis, as well as depletion of clotting factors, leading to diffuse clotting and hemorrhage. d. an inherited predisposition causes a deficiency of clotting factors that leads to overstimulation of coagulation processes in the vasculature.

c. a disease process stimulates coagulation processes with resultant thrombosis, as well as depletion of clotting factors, leading to diffuse clotting and hemorrhage.

A common site for the lesions associated with atopic dermatitis is the a. buttocks. b. temporal area. c. antecubital space. d. plantar surface of the feet.

c. antecubital space.

A patient is recovering from an uncomplicated MI. Which rehabilitation guideline is a priority to include in the teaching plan? a. refrain from sexual activity for a minimum of 3 weeks b. plan a diet program that aims for a 1-to 2-lb weight loss per week c. begin an exercise program that aims for at least 5-30 minute sessions per week d. consider the use of erectile agents and prophylactic NTG before engaging in sexual activity

c. begin an exercise program that aims for at least 5-30 minute sessions per week

Trends in the incidence and death rates of cancer include the fact that a. a higher percent of women than men have lung cancer. b. lung cancer is the most common type of cancer in men. c. blacks have a higher death rate from cancer than whites. d. breast cancer is the leading cause of cancer deaths in women.

c. blacks have a higher death rate from cancer than whites.

An expected finding in the assessment of an 81-year-old patient is a. a narrowed pulse pressure. b. diminished carotid artery pulses. c. difficulty isolating the apical pulse. d. an increased heart rate in response to stress.

c. difficulty isolating the apical pulse.

After a pneumonectomy, an appropriate nursing intervention is a. monitoring chest tube drainage and functioning b. positioning the patient on the unaffected side or back c. doing range-of-motion exercises on the affected upper limb d. auscultating frequently for lung sounds on the affected side

c. doing range-of-motion exercises on the affected upper limb.

In a severely anemic patient, the nurse would expect to find a. cyanosis and cardiomegaly. b. pulmonary edema and fibrosis. c. dyspnea at rest and tachycardia. d. ventricular dysrhythmias and wheezing.

c. dyspnea at rest and tachycardia.

Postoperatively, a patient who has had a laser prostatectomy has continuous bladder irrigation with a 3-way urinary catheter with a 30-mL balloon. When he reports bladder spasms with the catheter in place, the nurse should a. deflate the balloon to 10 mL to decrease bulk in the bladder. b. deflate the balloon and then reinflate to ensure that the catheter is patent. c. explain that this feeling is normal and that he should not try to urinate around the catheter. d. stop the irrigation, assess the patient's vital signs, and notify the HCP of possible obstruction.

c. explain that this feeling is normal and that he should not try to urinate around the catheter.

During the respiratory assessment of an older adult, the nurse would expect to find (select all that apply). a. a vigorous reflex cough b. increased chest expansion c. increased residual volume d. decreased lung sounds at base of lungs e. increased anteroposterior (AP) chest diameter

c. increased residual volume d. decreased lung sounds at base of lungs e. increased anteroposterior (AP) chest diameter

The most common finding in people at risk for sudden cardiac death is a. aortic valve disease b. mitral valve disease c. left ventricular dysfunction d. atherosclerotic heart disease

c. left ventricular dysfunction

The nurse is aware that a major difference between Hodgkin's lymphoma and non-Hodgkin's lymphoma is that a. Hodgkin's lymphoma occurs only in young adults. b. Hodgkin's lymphoma is considered potentially curable. c. non-Hodgkin's lymphoma can manifest in multiple areas. d. non-Hodgkin's lymphoma is treated only with radiation therapy.

c. non-Hodgkin's lymphoma can manifest in multiple areas.

In planning care for the patient with Crohn's disease, the nurse recognizes that a major difference between ulcerative colitis and Crohn's disease is that Crohn's disease a. often results in toxic megacolon. b. causes fewer nutritional deficiencies than ulcerative colitis. c. often recurs after surgery, while ulcerative colitis is curable with a colectomy. d. is manifested by rectal bleeding and anemia more often than is ulcerative colitis.

c. often recurs after surgery, while ulcerative colitis is curable with a colectomy.

The nurse identifies the flail chest in a trauma patient when a. multiple rib fractures are determined by x-ray b. a tracheal deviation to the unaffected side is present c. paradoxical chest movement occurs during respiration d. there is a decreased movement of the involved chest wall

c. paradoxical chest movement occurs during respiration

A patient with diabetes has a serum glucose level of 824 mg/dL (45.7 mmol/L) and is unresponsive. After assessing the patient, the nurse suspects diabetes-related ketoacidosis rather than hyperosmolar hyperglycemia syndrome based on the finding of a. polyuria. b. severe dehydration. c. rapid, deep respirations. d. decreased serum potassium.

c. rapid, deep respirations.

A patient with a tricuspid valve disorder has impaired blood flow between the a. vena cava and right atrium b. left atrium and left ventricle c. right atrium and right ventricle d. right ventricle and pulmonary artery

c. right atrium and right ventricle

The nurse should be alert for which manifestations in a patient receiving a loop diuretic? a. restlessness and agitation b. paresthesias and irritability c. weak, irregular pulse and poor muscle tone d. increased blood pressure and muscle spasms

c. weak, irregular pulse and poor muscle tone

cannot have this 24 hours before a nuclear stress test

caffeine

Without insulin, glucose

cannot get inside the cell to be used to make energy

What is the machine used during CABG to oxygenate and pump blood?

cardiopulmonary bypass machine

What type of surgeon performs a CABG?

cardiothoracic surgeon

Diabetic Ketoacidosis

cell starvation = liver releases glycogen; ketones and glucose in blood are high; strong ketones are strong acids...metabolic acidosis pH <7.3, sodium bicarb of <15, ketones in urin, blood glucose >250

A malignant neoplasm (cancer) has

cells with abnormal growth patterns, multiple functions, and the ability to spread to distant sites.

Diabetic retinopathy

changes in the capillary beds in the retina decreasing blood supply; causes ischemia (ultimately leads to blindness)

A patient who comes to the clinic reports frequent, watery stools for 2 days. Which action should the nurse take first?

check the patients blood pressure

pulmonary embolism (PE)

clot or other material lodges in vessels of the lung

Hydramnios

condition in which there is too much amniotic fluid around the fetus

during a PCI a stent may be placed here

coronary arteries

What does CABG stand for?

coronary artery bypass graft

A priority consideration is the management of the older adult with hypertension is to a. Prevent primary hypertension from converting to secondary hypertension b. Recognize that the older adult is less likely to adhere to the drug therapy regimen than a younger adult c. Ensure that the patient receives larger initial doses of antihypertensive drugs because of impaired absorption d. Use precise technique in assessing the BP of the patient because of the possible presence of an auscultatory gap.

d. Use precise technique in assessing the BP of the patient because of the possible presence of an auscultatory gap.

A patient asks, "How does air get into my lungs?" The nurse bases her answer on the knowledge that air moves into the lungs because of a. positive intrathoracic pressure b. contraction of the accessory abdominal muscles c. stimulation of the respiratory muscles by the chemoreceptors d. a decrease in intrathoracic pressure from an increase in thoracic cavity size

d. a decrease in intrathoracic pressure from an increase in thoracic cavity size

A nursing intervention that is most appropriate to decrease postoperative edema and pain after an inguinal herniorrhaphy is to a. apply a truss to the hernia site. b. allow the patient to stand to void. c. support the incision during coughing. d. apply a scrotal support with an ice bag.

d. apply a scrotal support with an ice bag.

The most common type of leukemia in adults in western countries is a. acute myelocytic leukemia. b. acute lymphocytic leukemia. c. chronic myelocytic leukemia. d. chronic lymphocytic leukemia.

d. chronic lymphocytic leukemia.

Because myelodysplastic syndrome arises from the pluripotent hematopoietic stem cell in the bone marrow, laboratory results the nurse would expect to find include a(n) a. excess of T cells. b. excess of platelets. c. deficiency of granulocytes. d. deficiency of all cellular blood components.

d. deficiency of all cellular blood components.

The nurse would expect that a patient with von Willebrand disease undergoing surgery would be treated with administration of vWF and a. thrombin. b. factor VI. c. factor VII. d. factor VIII.

d. factor VIII.

The nurse is unable to flush a central venous access device and suspects occlusion. The best nursing intervention would be to a. apply warm moist compresses to the insertion site b. try to force 10 mL of normal saline into the device c. place the patient on the left side with the head down d. have the patient change positions, raise arm, and cough

d. have the patient change positions, raise arm, and cough

The nurse is caring for a 59-year-old woman who had surgery 1 day ago to remove an ovarian cancer mass. The patient is awaiting the pathology report. She is tearful and says that she is scared to die. The most effective nursing intervention at this point is to use this opportunity to a. motivate change in an unhealthy lifestyle. b. teach her about the 7 warning signs of cancer. c. discuss healthy stress relief and coping practices. d. let her communicate about the meaning of this experience.

d. let her communicate about the meaning of this experience.

To decrease the patient's discomfort related to discussing his reproductive organs, the nurse should a. relate his sexual concerns to his sexual partner. b. arrange to have male nurses care for the patient. c. give him written material and ask if he has questions. d. maintain a nonjudgmental attitude toward his sexual practices.

d. maintain a nonjudgmental attitude toward his sexual practices.

The nurse counsels the patient receiving radiation therapy or chemotherapy that a. effective birth control methods should be used for the rest of the patient's life. b. after successful treatment, patients can expect a return to their previous level of function. c. the cycle of fatigue-depression-fatigue that may occur during treatment may be reduced by restricting activity. d. nausea and vomiting can usually be managed with antiemetic drugs, diet modification, and other interventions.

d. nausea and vomiting can usually be managed with antiemetic drugs, diet modification, and other interventions.

A characteristic of the stage of progression in cancer development is a. oncogenic viral transformation of target cells. b. a reversible steady growth facilitated by carcinogens. c. a period of latency before clinical detection of cancer. d. proliferation of cancer cells despite host control mechanisms.

d. proliferation of cancer cells despite host control mechanisms.

A patient has the following arterial blood gas results: pH 7.52, PaCO2 of 30mmhg, HCO3 of 24 mEq/L. The nurse determines these results indicate a. metabolic acidosis b. metabolic alkalosis c. respiratory acidosis d. respiratory alkalosis

d. respiratory alkalosis

To prevent the infection and transmission of STIs, the nurse's teaching plan would include an explanation of a. the appropriate use of oral contraceptives. b. the need for annual Pap tests for women with HPV. c. sexual positions that can be used to avoid infection. d. sexual practices that are considered high-risk behaviors.

d. sexual practices that are considered high-risk behaviors.

During the postoperative care of a 76-year-old patient, the nurse monitors the patient's intake and output carefully, knowing that the patient is at risk for fluid and electrolyte imbalances primarily because a. older adults have an impaired thirst mechanism and need reminding to drink fluids b. water accounts for a greater percentage of body weight in the older adult than in younger adults c. older adults are more likely than younger adults to lose extracellular fluid during surgical procedures d. small losses of fluid are significant because body fluids account for 45% to 50% of body weight in older adults

d. small losses of fluid are significant because body fluids account for 45% to 50% of body weight in older adults

An appropriate nursing intervention to assist a patient with pneumonia manage thick secretions and fatigue would be to a. perform postural drainage every hour b. provide analgesics as ordered to promote patient comfort c. administer O2 as prescribed to maintain optimal O2 levels d. teach the patient how to cough effectively and expectorate secretions

d. teach the patient how to cough effectively and expectorate secretions

Multiple drugs are often used in combinations to treat leukemia and lymphoma because a. there are fewer toxic and side effects. b. the chance that one drug will be effective is increased. c. the drugs are more effective without causing side effects. d. the drugs work by different mechanisms to maximize killing of cancer cells.

d. the drugs work by different mechanisms to maximize killing of cancer cells.

The spouse of a client who had coronary artery bypass graft (CABG) surgery asks why there is a dressing on the client's left leg. Which explanation would the nurse give?

"A vein in the left leg was used to bypass the coronary artery"

Which instructions about the use of nitroglycerin to prevent angina will the nurse provide to a client?

"Before physical activity, place one tablet under the tongue, and repeat the does in 5 minutes if pain occurs"

A 30-year-old client who is diagnosed with hyperlipidemia and hypertension asks the nurse to explain why treatment is important, stating "I feel fine, so I don't really see the need to make any changes." Which response would the nurse make?

"Both high blood pressure and high cholesterol contribute to development of heart disease"

Which instruction is beneficial for an aging African-American client with hypertension?

"Have an annual urinalysis"

The nurse has telephone messages from four patients who requested information and assistance. Which one should the nurse refer to a social worker or community agency first?

"I ran out of money and am cutting my insulin dose in half"

Sublingual nitroglycerin is prescribed for a client with a history of a myocardial infarction and atrial tachycardia. The nurse instructs the client about the prophylactic use of these tablets. Which statement by the client indicates the teaching was effective?

"I should take one tablet before attempting activity that has caused angina"

A registered nurse is teaching a nursing student about when a client with high blood pressure would follow up with the primary health care provider. Which statement made by the nursing student indicates effective learning?

"I will advise a client with a blood pressure of 130/80 mm Hg to follow up in a year"

After the nurse has completed discharge teaching for a client who has had a myocardial infarction, which client statement indicates that more teaching is needed?

"I will avoid physical activity"

A client is precepting a new graduate nurse and the new graduate is assigned to care for a client with chronic pain. Which statement, if made by the new graduate nurse, indicates the need for further teaching regarding pain management?

"I will be sure to cue in to any indicators that the client may be exaggerating their pain"

Spironolactone, an aldosterone antagonist, is prescribed for a patient. Which statement by the patient indicates that the teaching about this medication has been effective?

"I will drink apple juice instead of orange juice for breakfast"

The patient had diarrhea for 5 days and developed an acid-base imbalance. Which statement would indicate that the nurse's teaching about the acid-base imbalance has been effective?

"My blood became too acid because I lost some base in the diarrhea fluid"

Which response by the nurse is best when a client who has had an ST segment elevation myocardial infarction (STEMI) asks about the resumption of sexual activity?

"Sexual activities can be safely resumed after an exercise stress test with no heart symptoms"

When a client who is scheduled to have coronary artery bypass graft surgery asks the nurse what benefit can be expected from the surgery, which response would the nurse make?

"This surgery significantly decreases symptoms in most clients"

Which statements said by patients indicate that the nurses teaching regarding prevention of acid-base imbalances is successful? SATA

-"I should take my insulin on time every day" -"My aspirin is on high shelf away from children" -"I have reliable transportation to dialysis sessions"

A 46-year-old client was diagnosed with a URI yesterday and antibiotics were prescribed in the ED. Has history of IDDM, HTN, peripheral neuropathy, vascular dx., and retinopathy. Initial nursing shift assessment findings: --- alert and oriented --- productive cough --- serum glucose = 486 mg/dl --- hgb AIC = 6.8% --- serum potassium 3.5 mEq/L --- rhinitis with pale yellow, thick drainage --- reports urinating several times an hour --- burning pain from toes to calves bilaterally

--- serum glucose = 486 mg/dl --- serum potassium 3.5 mEq/L --- reports urinating several times an hour --- burning pain from toes to calves bilaterally

Nitroglycerin sublingual tablets are prescribed for a client with the diagnosis of angina. The nurse advises the client to anticipate pain relief will begin within which period of time?

1 to 3 minutes

Glipizide

10 mg orally daily; class: second generation sulfonylurea; action: promotes insulin secretion in pancreas

Carb load before exercising; it's recommended that athletes and kids get in

15-30 grams in kids and athletes

How many IV's are needed for a PCI?

2 IV's preferred in the left arm

Glucose < 60 (70) adult treatment =

20-50 g of D50W via IV

Metoprolol

25 mg oral exR daily; class: beta-adrenergic blocker; lowers blood pressure

Glycosylated hemoglobin (A1C) checks

3 months average glucose

Gabapentin

300 mg orally daily; class: antiepileptic; action: relieves neuropathic pain

Exenatide

5 mcg subq injection twice daily; class: incretin mimetic; action: slows gastric emptying, stimulates insulin release, suppresses glucagon release, decreases appetite

The pancreas is

5% endocrine and 95% exocrine

What is the normal cardiac output volume?

5-6L

Metformin

500mg oral exR daily; class: biguanide; action: decreases glucosa production by the liver and increases tissue response to insulin

Normal glycemic control in adults

60-100

Which of these clients seen at a health fair will be most at risk for hypertension?

62-year-old African American man

Aspirin

81mg daily;class: salicylate; action: minimizes platelet aggregation

How many minutes from door to balloon?

90 minutes

What is the ABCDE method?

A= asymmetrical shape; B= irregular border; C= changes in color; D= diameter greater than ¼; E= evolving; changes over time

Which finding in a client with a diagnosis of stable angina is most important for the nurse to communicate to the health care provider?

Angina episodes are occurring more frequently

When caring for a client who has just arrived in the emergency department with possible acute coronary syndrome, which prescribed action would the nurse take first?

Ask the client about the level of intensity of the chest pain

Which nursing action has the highest priority when caring for a client receiving nitroglycerin for the treatment of angina?

Asking the client to sit or stand slowly

A client who is being treated for uncontrolled hypertension develops a nosebleed. After applying pressure to the nose, which action would the nurse take next?

Assess the client's blood pressure

Which nursing action is a priority of care provided to clients with chest pain?

Assessing airway-breathing-circulation (ABC)

A client is receiving radiation therapy internally. How far should visitors be from client?

At least 6 feet from the client

Which clinical condition will result in changes in the integrity of the arterial walls and small blood vessels?

Atherosclerosis

Why can't long-acting insulins be drawn up with other medicines?

Because of the pH in them

Why are metformin and glyburide not recommended as first-line treatment for GDM?

Because they are known to cross the placenta

A client is admitted to the hospital with a long history of uncontrolled hypertension. Which laboratory result will be most important for the nurse to review?

Blood urea nitrogen

The nurse can best determine adequate arterial oxygenation of the blood by assessing: A. heart rate B. hemoglobin level C. arterial oxygen tension D. arterial carbon dioxide tension

C. arterial oxygen tension

Which type of leukemia is linked to the genetic change called the Philadelphia chromosome in 20% of clients, low RBC, HGB, platelet, and HCT?

CLL (Chronic Lymphocytic Leukemia)

CLL is 20%

CLL is 20%

What are symptoms of severe hypoglycemia?

CVA like symptoms, altered consciousness, coma, and death

What does diabetes do?

Cause microvascular changes

What is a dysplasia of cells?

Cells have an abnormal shape, size, or appearance

7 warning signs of cancer

Change in bowel or bladder habits; a sore that doesn't heal; unusual bleeding or discharge; thickening or lump in the breast or elsewhere in the body; constant indigestion or difficulty in swallowing; obvious change in a wart or mole; nagging cough or hoarseness

A client with known peripheral arterial disease calls the clinic and tells the nurse about experiencing several symptoms. Which symptom requires the most rapid action by the nurse?

Chest pain

A client with long-term cocaine use presents with extreme suspiciousness. Which additional clinical manifestations would the nurse monitor for? SATA

Chest pains Panic attacks Nasal damage

Always draw up ____ insulin first.

Clear (because if mixed, detrimental effects to client)

Which information about a client who is being discharged 3 days after having an ST segment elevation myocardial infarction (STEMI) and coronary artery stent placement indicates that a home health referral may be needed at discharge?

Client reports frequently forgetting to take medications

Which client in the emergency department would the nurse assess first?

Client with chest pressure and ST segment elevation on the electrocardiogram

Which client would be at an increased risk for coronary artery disease (CAD)? (cholesterol) SATA

Client with total cholesterol 250 mg/dL and LDL cholesterol 120 mg/dL

Which finding about a client's angina is most important for the nurse to communicate to the health care provider?

Continues after rest and nitroglycerin

What are the main symptoms of lung cancer?

Coughing (including coughing up blood), weight loss, shortness of breath, chest pain, hoarseness, bone pain, headache

diet recommended for patients with HTN

DASH

A health care provider prescribes morphine for a client being treated for myocardial infarction. Which physiological response will occur if the client experiences the intended therapeutic effect of morphine?

Decreased workload of the heart

Paradoxical chest movement

Describes an abnormal chest movement, with your chest moving inward during inhalation rather than outward

In the TMN classification system what does T1, T2, T3, T4 mean?

Describes primary tumor based on size and or invasion into nearby structures; the higher the T number, the larger the tumor and or the more it has grown into nearby sites

In the TMN classification system what does M1, M2, M3, M4 mean?

Describes the extent of metastasis; the higher the M number, the more extensive the metastasis

In the TMN classification system what does N1, N2, N3 mean?

Description of size, location, and or number of lymph nodes involved; the higher the N number, the more extensive the lymph node involvement

Bilateral and changes to sensation = _____ while unilateral and no changes = _____.

Diabetes; spinal injury

A client is receiving metoprolol. Which potential side effect will the nurse teach the client to expect?

Dizziness with strenuous activity

A client with hypertension has received a prescription for metoprolol. Which information will the nurse include when teaching this client about metoprolol?

Do no abruptly discontinue the medication

stress test that requires no beta-blockers for 24 hours

Dobutamine

When teaching a group of female clients about the clinical manifestations of coronary artery disease that are more common in women, which symptoms would the nurse include?

Dyspnea Indigestion Unusual fatigue

1st diagnostic for a client with CP

EKG

When an obese client receives a diagnosis of high blood pressure, which topic would be the most important to include in client teaching?

Effect of weight loss in hypertension

When planning discharge teaching for a client who had coronary artery bypass graft (CABG) surgery using a vein graft, which information will the nurse include?

Elevate the leg that provided the vein graft whenever you are sitting

When caring for a client who presents to the emergency department with an ST-segment-elevation myocardial infarction (STEMI), which laboratory result will the nurse expect?

Elevated serum troponin I and T

After a client has had a cardiac catheterization, which finding requires the most rapid action by the nurse?

Heart rate 114 beats/minute

When planning care for a client who has just returned to the nursing unit after placement of a coronary artery stent that was accomplished via access through the femoral artery, which complication is a priority for the nurse to prevent?

Hematoma formation

What are the 2 types of lymphoma?

Hodgkin's and Non-Hodgkin's

The health care provider prescribes atenolol for a client with angina. Which potential side effect will the nurse mention when instructing the client about this medication?

Hypotension

Diabetic Ketoacidosis treatment

IVF: 15-20 ml/kg bolus of Normal Saline (repeat PRN); regular insulin bolus, then continuous drip; correct electrolytes; cardiac monitoring

Nursing Diagnosis of TB

Impaired breathing Impaired airway clearance Risk for infection Lack of knowledge

How often do you recheck BG when hypoglycemic episode has occurred?

In 10-15 minutes; do this repeatedly if altered LOC

What are causes of diabetic ketoacidosis?

Infection, diet, stress, no insulin or meds

While in the postanesthesia care unit, a client reports shortness of breath and chest pain. Which is the most appropriate initial response by the nurse? 1 Initiate oxygen via a nasal cannula 2 Administer the prescribed morphine 3 Prepare the client for endotracheal intubation 4 Place a nitroglycerin tablet under the client's tongue

Initiate oxygen therapy via a nasal cannula

What are some early signs and symptoms of breast cancer?

Inverted nipple, nipple discharge, lump or nodule in breast, increase in size or shape of breast, breast tenderness, swelling, redness or skin changes or pain in breast.

Which information would the nurse include when explaining the purpose of a thallium scan to the client who has a history of chest pain?

It assesses myocardial ischemia and perfusion

When assessing a client's blood pressure, the nurse notes that the diastolic blood pressure reading in the right arm is 10 mm Hg higher than the blood pressure in the left arm. Which statement reflects the nurse's understanding of this occurrence?

It is a normal occurrence

A patient with DM has an ABG results: pH=7.28, PaCO2=34, PaO2=85, HCO3=18. The nurse would expect which finding?

Kussmaul respirations

GDM can cause

LGA (large gestational age), heart failure, GI problems, infection in infant, hydramnios

What does T-4, N-1, Mx mean?

Large tumor, single node involvement and unable to assess metastasis

Which dietary choices would the nurse teach for the client with peripheral artery disease? SATA

Limit salt intake Eat whole-grain breads Use liquid vegetable oils Avoid processed meats

When the nurse is caring for a client who is 90 minutes postcoronary stenting through the femoral artery, which assessment finding would prompt the nurse to activate the rapid response team?

Loss of pedal pulse on the affected limb

Hypoglycemia is experienced with

MOA inhibitors, Inderal, and salicylates

In the TMN classification system what does MX mean?

Metastases cannot be evaluated

Is this right? Type 1 diabetics and children under 10 years old get insulin while children 10 years or older get

Metformin

Basel cell carcinoma

Most common and least severe type of skin cancer

Intermediate insulin

NPH - onset 1-3 hours; peaks at 5-8 hours; lasts 14-18 hours

When a client comes to the emergency department with moderate substernal chest pain that is unrelieved by rest and nitroglycerin, which prescribed action has the highest priority?

Obtain a 12-lead electrocardiogram (ECG)

Which information obtained by the nurse about a client would represent risk factors for the client's admission diagnosis of hypertension? SATA

Occasional cocaine use African American heritage

What age group does cancer most likely occur?

Over 50 years of age

What is the first line of treatment for a STEMI?

PCI or heart cath

What is the most important aspect to treat when a patient has Xerostomia?

Pain

What are the symptoms of Non-Hodgkin's Lymphoma (NHL)?

Painless lymph node enlargement; NHL can originate outside lymph nodes so the method of spreading is unpredictable, and may have widely disseminated disease by time detected; Hepatomegaly with liver involvement, neuro symptoms with CNS disease, airway obstruction, hyperuricemia and renal failure from tumor lysis syndrome, pericardial tamponade, and GI complaints

A client with myocardial infarction is brought to the emergency department, and the primary health care provider recommends the placement of a stent. The client is incompetent to understand the situation. Which model would the nurse manager think would be beneficial in this situation?

Patient-benefit model

CML =

Philadelphia chromosome is present in 90-95% of cases

Classic symptoms for all types of diabetes include:

Polydipsia (thirst); Polyuria (increased urination); polyphagia (increased appetite)

What are the 4 types of diabetes?

Primary, Type I 'juvenile; Secondary, Type II, Gestational, and metabolic syndrome

The nurse is caring for a client who is receiving aspirin therapy. Which clinical indicator would be related to this therapy?

Prolonged bleeding time

A client enters the emergency department reporting shortness of breath and epigastric distress. Which would be the triage nurse's first intervention?

Provide the client with oxygen

Which intervention would the nurse perform when caring for a client in the emergency department reporting chest pain? SATA

Providing oxygen Assessing vital signs Obtaining a 12-lead EKG Drawing blood for cardiac enzymes Auscultating heart sounds Administering nitroglycerin

What is brachytherapy?

Radioactive seeds implanted into the prostate to treat prostate cancer

*NTK Which types of insulin require you have a meal tray nearby?

Rapid acting (Humalog or Novolog) and regular insulin

When caring for a client who is hospitalized for an acute myocardial infarction, which prescription by the health care provider would the nurse question?

Rectal suppository as needed for constipation

What are the giant, multi-nucleated cells called that are located in the lymph nodes associated with Hodgkin's lymphoma?

Reed-Stenberg cells

What is the main diagnostic feature in Hodgkin's Lymphoma?

Reed-Stenberg cells

After obtaining client blood pressures of 172/104 mm Hg and 164/98 mm Hg during a blood pressure screening, which action would the nurse take next?

Refer the client to a primary health care provider

The nurse has administered sublingual nitroglycerin. Which outcome would the nurse use to determine the effectiveness of sublingual nitroglycerin?

Relief of anginal pain

A client in the coronary care unit develops "viselike" chest pain radiating to the neck. Assessment reveals a blood pressure of 124/64 mm Hg, an irregular apical pulse of 64 beats per minute, and diaphoresis. Cardiac monitoring is instituted, and morphine sulfate 4 mg intravenous (IV) push stat is prescribed. Which intervention is the priority nursing care for this client?

Relief of pain

Which pain characteristic would the nurse expect to observe when a client is experiencing anginal pain?

Relieved by sublingual nitroglycerin

What is a wedge resection of the lung?

Removal of a small wedge-shaped piece of lung tissue to remove a small tumor to diagnose (dx) lung cancer

When the nurse is obtaining a health history for a client scheduled for cardiac catheterization, which client information is most important to communicate to the health care provider?

Reports allergy to most shellfish

What are the symptoms of Diabetic Ketoacidosis?

Same as hyperglycemia, but more extreme, nausea, vomiting, dehydration, vision changes, Kussmaul respirations (rapid dynamic pattern that's trying to correct acidosis)

What are at least 2 types of breast cancer screening?

Self-breast exam, mammogram, ultrasound, MRI, breast biopsy

Which clinical manifestations are more likely to occur in women with coronary artery disease compared with men? SATA

Severe fatigue Sense of unease Shortness of breath

Which clinical finding would the nurse expect for a client with hypertensive emergency?

Severe pounding headache

When a client is admitted to the coronary care unit with a diagnosis of ST segment elevation myocardial infarction, how will the nurse expect to describe the pain?

Severe, intense chest pain

Which assessment finding indicates a need for the nurse to consult with the health care provider before administering the prescribed metoprolol to a client with stable angina?

Sinus bradycardia, rate of 54 on monitor

After consistently obtaining a blood pressure of 140/76 mm Hg for a client, which stage of hypertension will the nurse document?

Stage 2

How is inulin administered?

Subcutaneously

What are symptoms of moderate hypoglycemia?

Sweating, tremors, confusion, slurred speech

Type 1 diabetes is caused by

T cell mediated autoimmune destruction of the beta cells

When a client is diagnosed with microvascular angina, which topics would the nurse include in client teaching? SATA

Taking daily dose of aspirin Smoking cessation Taking nitroglycerin

When a client with angina is scheduled to have a cardiac catheterization, which explanation would the nurse give about the purpose of the procedure?

To visualize the disease process in the coronary arteries

Which laboratory test is most important for the nurse to monitor when a client is admitted with acute coronary syndrome?

Troponin

Which laboratory value will be most important for the nurse to monitor to determine whether a client with chest pain has acute coronary syndrome (ACS)?

Troponin T (cTnT)

In the TMN classification system what does TX mean?

Tumor cannot be measured

In the TMN classification system what does Tis mean?

Tumor in situ, meaning malignant cells only within superficial layer of tissue; no extension into deeper tissue

Anatomic extent of disease is based on what 3 parameters?

Tumor size and invasiveness (T); Spread to lymph nodes (N); Metastasis (M) i.e. TMN classification

Low or absent c-peptide concludes which type of diabetes?

Type 1

When discharging a client who has had insertion of a coronary artery stent, the nurse will instruct the client to seek immediate medical attention for which signs and symptoms? SATA

Unexplainable profuse diaphoresis Indigestion not relieved by antacids Acute chest pain after rigorous exercise Continued chest pain after nitroglycerin use

After the nurse teaches a group of women about coronary artery disease (CAD) and myocardial infarction (MI), which statement by the women indicates that the teaching has been effective?

Unusual fatigue is a common symptom of CAD in women

Which finding in a client who has been admitted with myocardial infarction is most important to communicate to the health care provider?

Urine output 15 mL/h

Which topics would the nurse include in teaching a client with a new diagnosis of hypertension? SATA

Use of a home blood pressure monitor Adverse effects of tobacco on blood pressure Benefits of moderate daily exercise

Which activities might cause chest pain in a client with stable angina? SATA

Walking outside on a cold day Sexual activity Smoking a cigarette Use of an oral decongestant

Nursing management - Nursing implications on reactions in the oral mucosa =

Xerostomia

Metabolic syndrome is

a cluster of conditions that occur together, increasing risk of heart disease, stroke, and type 2 diabetes

Sometimes diabetes is caused by

a virus or traumatic event

The nurse determines a patient undergoing ileostomy surgery understands the procedure when the patient states a. "I should only have to change the pouch every 4 to 7 days." b. "The drainage in the pouch will look like my normal stools." c. "I may not need to wear a drainage pouch if I irrigate it daily." d. "Limiting my fluid intake should decrease the amount of output."

a. "I should only have to change the pouch every 4 to 7 days."

Nursing interventions for a patient with severe anemia related to peptic ulcer disease include (select all that apply) a. instructions for high-iron diet. b. taking vital signs every 8 hours. c. monitoring stools for occult blood. d. teaching self-injection of erythropoietin. e. administration of cobalamin (vitamin B12) injections.

a. instructions for high-iron diet. c. monitoring stools for occult blood.

During administration of a hypertonic IV solution, the mechanism involved in equalizing the fluid concentration between ECF and the cells is: a. osmosis b. diffusion c. active transport d. facilitated diffusion

a. osmosis

What is known as the amount of resistance within the vasculature that the heart pumps against it?

afterload

A client with a 3-day history of nausea and vomiting presents to the emergency department. The client is hypoventilating and has a respiratory rate of 10 breaths per minute. The electrocardiogram (ECG) monitor displays tachycardia, with a heart rate of 120 beats per minute. ABGs are drawn and the nurse reviews the results, expecting to note which finding?

an increased pH and an increased HCO3

Part of a PCI that requires contrast dye

angiogram

During a PCI what radiologic intervention is used?

angiogram or arteriogram

the use of a balloon to unblock a coronary artery

angioplasty

Xerostomia causes

anorexia, loss of taste, aversion to food, and erythema

RX prevent occlusion of newly placed stent

antiplatelet

A patient who has a history of COPD was hospitalized for increasing shortness of breath and chronic hypoxemia. In planning for discharge, which action by the nurse will be most effective in improving compliance with discharge teaching?

arrange for the patients caregiver to be present during the teaching

Rapid acting insulin

aspart, glulisine, lispro - onset 10-15 minutes; peaks at 1-1.5 hours; lasts 3-5 hours

NSAID given at the first sign of a MI

aspirin

Colonoscopy should be received

at age 50 if no family history

When caring for a patient with lung abscess, what is the nurses priority intervention? a. Postural drainage b. Antibiotic administration c. obtaining a sputum specimen d. Patient teaching about home care

b. Antibiotic administration

What is the most frequent form of skin cancer?

basal cell carcinoma

Both clopidogrel and aspirin prevents platelet aggregation and have a common side effect of

black tarry stools or bleeding gums

Nursing implications - common side effects of cancer treatment

bone marrow suppression (most common); fatigue and shortness of breath; GI disturbances; integumentary and mucosal reactions; pulmonary effects; reproductive effects

A patient with stage I colorectal cancer is scheduled for surgery. Patient teaching for this patient would include an explanation that a. chemotherapy will begin after the patient recovers from the surgery. b. both chemotherapy and radiation can be used as palliative treatments. c. follow-up colonoscopies will be needed to ensure that the cancer does not recur. d. a wound, ostomy, and continence nurse will visit the patient to identify the site for the ostomy.

c. follow-up colonoscopies will be needed to ensure that the cancer does not recur.

A patient with multiple myeloma becomes confused and lethargic. The nurse would expect that these clinical manifestations may be explained by diagnostic results that indicate a. hyperkalemia. b. hyperuricemia. c. hypercalcemia. d. CNS myeloma.

c. hypercalcemia.

The typical fluid replacement for the patient with a fluid volume deficit is a. dextran b. 0.45 saline c. lactated Ringer's solution d. 5% dextrose in 0.45% saline

c. lactated Ringer's solution

pleurodesis

creation of adhesions between the parietal and visceral pleura to treat recurrent pneumothorax

Glucose is too large of a molecule to

cross the cell membrane without the help of insulin

Which skills can the nurse delegate to assistive personnel (AP)? (Select all that apply.) a. Initiate oxygen therapy via nasal cannula. b. Perform nasotracheal suctioning of a patient. c. Educate the patient about the use of an incentive spirometer. d. Assist with care of an established tracheostomy tube. e. Reposition a patient with a chest tube.

d. Assist with care of an established tracheostomy tube. e. Reposition a patient with a chest tube.

Which respiratory assessment finding does the nurse interpret as abnormal? a. Inspiratory chest expansion of 1 inch b. Symmetric chest expansion and contraction c. Resonance (to percussion) over the lung bases d. Bronchial breath sounds in the lower lung fields

d. Bronchial breath sounds in the lower lung fields

What is the priority action for the nurse to take if the patient with type 2 diabetes reports blurred vision and irritability? a. Call the provider. b. Give insulin as ordered. c. Assess for other neurologic symptoms. d. Check the patient's blood glucose level.

d. Check the patient's blood glucose level.

A patient on chemotherapy for 10 weeks started at a weight of 121 lb. She now weighs 118 lb and has no sense of taste. Which nursing intervention would be a priority? a. Discuss with the provider the need for parenteral nutrition. b. Teach the patient to eat foods that are fatty, fried, or high in calories. c. Tell the patient to drink a nutritional supplement beverage three times a day. d. Have the patient try various spices and seasonings to enhance the flavor of food.

d. Have the patient try various spices and seasonings to enhance the flavor of food.

In teaching a patient with hypertension about controlling the illness, the nurse recognizes that a. All patients with elevated BP need drug therapy b. Obese persons must achieve a normal weight to lower bp c. It is not necessary to limit salt in the diet if taking a diuretic d. Lifestyle modifications are needed for all persons with elevated bp

d. Lifestyle modifications are needed for all persons with elevated bp

What should a patient be taught after a hemorrhoidectomy? a. Take mineral oil before bedtime. b. Eat a low-fiber diet to rest the colon. c. Use oil-retention enemas to empty the colon. d. Take prescribed pain medications before a bowel movement.

d. Take prescribed pain medications before a bowel movement.

Which statement would be correct for a patient with type 2 diabetes who was admitted to the hospital with pneumonia? a. The patient must receive insulin therapy to prevent ketoacidosis. b. The patient has islet cell antibodies that have destroyed the pancreas's ability to make insulin. c. The patient has minimal or absent endogenous insulin secretion and requires daily insulin injections. d. The patient may have enough endogenous insulin to prevent ketosis but is at risk for hyperosmolar hyperglycemia syndrome.

d. The patient may have enough endogenous insulin to prevent ketosis but is at risk for hyperosmolar hyperglycemia syndrome.

Explain to the patient with gonorrhea that treatment will include both ceftriaxone and azithromycin because a. azithromycin helps prevent recurrent infections. b. some patients do not respond to oral drugs alone. c. coverage with more than one antibiotic will prevent reinfection. d. the increasing rates of drug resistance requires using at least 2 drugs.

d. the increasing rates of drug resistance requires using at least 2 drugs.

Complications of transfusions that can be decreased by using leukocyte depletion or reduction of RBC transfusion are a. chills and hemolysis. b. leukostasis and neutrophilia. c. fluid overload and pulmonary edema. d. transmission of cytomegalovirus and fever.

d. transmission of cytomegalovirus and fever.

The most effective method of administering a chemotherapy agent that is a vesicant is to a. give it orally. b. give it intraarterially. c. use an Ommaya reservoir. d. use a central venous access device.

d. use a central venous access device.

A patient who has been hospitalized for 2 days has a nasogastric tube to low suction and is receiving normal saline IV at 100 mL/hr. Which assessment finding would be a priority for the nurse to report to the health care provider?

decreased alertness since admission

Long Acting

detemir; glargine - onset 1-2 hours; detemir peaks at 8-10 hours and lasts 12-24 hours, glargine does not peak and lasts 22-24 hours

Too little or no insulin =

diabetes

Creatinine:

diabetes causes macrovascular kidney changes. Stress on the nephrons filtering the glucose, starts leaking albumin. Glomeruli thicken. Function loss.

Hypoglycemia, Diabetic Ketoacidosis, and Hyperosmolar Hyperglycemic Non-Ketotic Syndrome are all

diabetic emergencies

What is the drug class that decreases pre-load by making you produce more urine?

diuretics

What kind of stent delivers therapeutic levels of medications locally to prevent cell proliferation?

drug eluting stent

An older adult patient who is malnourished presents to the emergency department with a serum protein level of 5.2. Which clinical manifestation should the nurse expect?

edema

SGLT drug that rids glucose through kidneys and bowel

empagliflozin

Hyperglycemia is experienced with

estrogens, K+ sparing diuretics, and glucocorticoids

injectable medication that increases insulin production, taken with meals, decreases appetite

exenatide

What is leukemia?

general term to describe a group of malignant disorders affecting the blood and blood-forming tissues of bone marrow, lymph system, and spleen

A patient who had a transverse colectomy for diverticulosis 18 hours ago has nasogastric suction. The patient reports anxiety and incisional pain. The patient's respiratory rate is 32 breaths/min, the ABGs indicate respiratory alkalosis. Which action should the nurse take first?

give the patient the PRN IV morphine sulfate 4mg

long acting insulin can be taken any time of day

glargine

oral sulfonylurea, take with meals, stimulates insulin production

glipizide

Decreased BG =

glucagon is released to stimulate liver, muscle cells to release glucose into blood

No insulin or low insulin means that

glucose cannot get in cell to be used for functioning

When screening for gene mutations

go back 2 generations for history

HHNS

has 10-20% mortality rate; severe hypovolemic shock; risk of severe hyponatremia with severe neurological dysfunction

*NTK When giving Novolog or Humalog (rapid acting insulin) you must

have meal tray at bedside (should have food near by with regular insulin too)

the physiological process that stops bleeding

hemostasis

Thrombolytic that is given when MI is suspected

heparin

Which thrombolytic can be given SQ or IV and the side effects are measured by the PTT?

heparin

Sudden, rapid, significant elevation of BP

hypertensive crisis

What is considered severe elevation of BP that is greater than 180 systolic or 110 diastolic?

hypertensive emergency

BG < 70 =

hypoglycemia - the brain is deprived of fuel

ICAs (islet cell antibodies), GAD (glutamic acid decarboxylase) antibody test, and C-Peptide tests are all

immunoassays that differentiate between type 1 and type 2 diabetes

A client with a 3-day history of nausea and vomiting presents to the emergency department. The client is hypoventilating and has a respiratory rate of 10 breaths/min. The ECG displays tachycardia, HR 120 bpm. ABGs are drawn and the nurse reviews the results, expecting to note which finding?

increased pH & increased HCO3

Carvedilol decreases the force of myocardial contraction and has a common side effect of

increased shortness of breath

The lungs as an acid-base buffer by:

increasing respiratory rate and depth when CO2 levels in the blood are high reducing acid load

A patient who had surgery for a perforated gastric ulcer has been receiving nasogastric suction for 3 days. The patient's serum sodium level is 127. Which prescribed therapy should the nurse question?

infuse 5% dextrose in water intravenously at 125 mL/hr

flail chest

instability of the chest wall resulting from trauma

Potassium:

insulin drives potassium into cells. Can lower serum glucose

Increased BG =

insulin is released to move into cells

Type 2 diabetes is caused by

insulin resistance of cell (fat cells) and pancreas being 'overworked'

Too much glucose in blood causes water to push into cells causing one to become _____ which is a form of hypovolemic shock. As blood glucose is corrected, K+ is pulled back into cells. This blood glucose correction can cause _____.

intravascularly dry; hypokalemia

What are the symptoms of mild hypoglycemia?

irritability, headache, nausea, and vomiting

26% of hydramnios

is caused by women with GDM

the end result of poor perfusion

ischemia

When a diabetic is sick, it is best for them to

keep takin insulin and sip 8-12 oz of fluid each hour

The patient has severe hyperthyroidism and will have surgery tomorrow. What assessment is most important for the nurse to perform in order to detect development of the highest risk of acid-base imbalance?

level of consciousness

rapid acting insulin with onset in 10-15 minutes

lispro

Hypoglycemia is the number one diabetic emergency because

low blood sugar kills brain cells which can cause stroke or death

Hodgkin's Lymphoma

malignant condition characterized by proliferation of abnormal, giant multi-nucleated cells, called Reed-Stenberg cells, which are in lymph nodes; occurs most frequently in 15-30-years-old and above 55n years old; long term survival exceeds 80% for all stages

95% of all diabetics have insulin resistance; lifestyle management can determine

management as to whether to utilize oral drugs or insulin

A nurse is assessing a newly admitted patient with chronic heart failure who forgot to take prescribed medications. The patient seems confused and short of breath with peripheral edema. Which assessment should the nurse complete first?

mental status

A patient who is lethargic and with deep, rapid respirations has the following ABG results: pH=7.32, PaO2=88, PaCO2= 35, HCO3=16. How should the nurse interpret these results?

metabolic acidosis

ABG:

metabolic acidosis. Caused by breakdown of fats cells to produce energy....ketones are very acidic

The nurse is caring for a client with a NG tube that is attached to low suction. The nurse monitors the client for manifestations of which disorder that the client is at risk for?

metabolic alkalosis

The nurse is caring for a client with a nasogastric tube that is attached to low suction. The nurse monitors the client for manifestations of which disorder that the client is at risk for?

metabolic alkalosis

a biguanide insulin sensitizer that reduces glucose release from liver

metformin

the level of sedation during PCI

moderate

Although this drug helps with pain, the main benefit is reduced oxygen demand during a MI?

morphine

given for CP to improve oxygenation/reduce pain

morphine

Triglycerides:

most common fat in body. Too many calories, stored into fats. Then when body needs energy with insulin resistance causing normal cellular pathway to be 'blocked'...triglycerides released.

Xerostomia is when

mucus membranes become irritated and injured from radiation and or chemo

What drug can be given every 5 minutes for up to 3 does for chest pain?

nitroglycerin

0.6-1.3

normal creatinine range

intermediate insulin with peak at 5-8 hours; afternoon snacks required

nph

Type 2 diabetes in children...insulin resistance

occurs with age, obesity, sedentary lifestyle, and family history. It often goes undetected until infection, severe symptoms, DKA (diabetic keto acidosis)

Common risk factors for prostate cancer:

older age- risk increases after the age of 50; rare before 45, diet, obesity, smoking, chemical exposure (Agent Orange), STI's, vasectomy

Vitals that are taken post PCI to check for bleeding

orthostatic

The nurse reviews the ABG results of a client with atelectasis. The nurse analyzes the results and analyzes the results and determines that the client is experiencing respiratory acidosis. Which result validates the nurses findings?

pH=7.25 PaCO2=50

The laboratory technician calls with ABG results on four patients. Which result is most important for the nurse to report immediately to the health care provider?

pH=7.31 PaO2=91 PaCO2=50 O2=96%

What are the 5 P's?

pain parasthesia paralysis pallor pulselessness

After receiving change-of-shift report, which patient should the nurse assess first?

patient with serum magnesium level of 1.1 who has tremors and hyperactive reflexes

a insulin sensitizer that causes increased glucose in muscles; causes weight gain and edema

pioglitazone

A patient in metabolic alkalosis is admitted to the emergency department and pulse oximetry (SpO2) indicates that the O2 saturation is 94%. Which action should the nurse expect to take next?

place the patient on high-flow oxygen

The nurse is caring for a client who is on a mechanical ventilator. ABG results indicate: pH=7.50, PaCO2=30. The nurse has determined that the client is experiencing respiratory alkalosis. Which laboratory value would most likely be noted in this condition?

potassium level of 3.0

What is known as the amount of blood in the ventricles at the end of diastole?

preload

What is known as chest pain due to vasospasms that can be caused by stimulant abuse?

prinzmetal or variant

What is the difference between Systolic and Diastolic pressure?

pulse pressure

type of insulin used in insulin IV drips

regular

Short acting insulin

regular (Humulin-R; Novolin grToronto) - onset 30-45 minutes; peaks at 2-3 hours; lasts 6.5 hours

pneumonectomy

removal of a lung

The nurse assesses a patient with COPD who has been admitted after increasing dyspnea over the past 3 days. Which finding is important for the nurse to report to the health care provider?

respirations are 36 breaths/minute

The nurse is caring for a client with several broken ribs. The client is most likely to experience what type of acid-base imbalance?

respiratory acidosis from inadequate ventilation

The nurse is caring for a client with severe broken ribs. The client is most likely to experience what type of acid base imbalance?

respiratory acidosis from inadequate ventilation

A client who is found unresponsive has ABGs drawn and the results are: pH=7.12, PaCO2=90, HCO3=22. The nurse interprets the results as indicating which condition?

respiratory acidosis without compensation

A patient has the following arterial blood gas results: pH=7.52, PaCO2=30, HCO3=24. The nurse determines that these results indicate:

respiratory alkalosis

A patient who was involved in a motor vehicle crash has had a tracheostomy placed to allow for continued mechanical ventilation. How should the nurse interpret the following ABG results: pH=7.48, PaO2=85, PaCO2=32, HCO3=25?

respiratory alkalosis

The nurse reviews the ABG results of a client and notes the following: pH=7.45, PaCO2=30, HCO3=20. The nurse analyzes these results as indicating which condition?

respiratory alkalosis, compensated

After a PCI with a femoral approach, there is a high risk for this complication that presents with back pain?

retroperitoneal bleed

a complication of a groin approach PCI

retroperitoneal bleed

What hypertension is related to a specific underlying cause?

secondary hypertension

A patient is admitted to the emergency department with severe fatigue and confusion. Laboratory studies are done. Which laboratory value will require the most immediate action by the nurse?

serum calcium is 18

What does the S is PQRST stand for?

severity

oral incretin stimulating agent

sitagliptin

A patient with renal failure is on a low phosphate diet. Which food should the nurse instruct an UAP to remove from the patients food tray?

skim milk

Bilateral and changes to sensation = diabetes while unilateral and no changes =

spinal injury

What is described as chest pain that occurs upon exertion?

stable angina

The most common type of CABG is done through an incision where?

sternum

lisinopril prevents the development of heart failure and protects kidney profusion and has a common side effect of

swelling of the tongue or lips

What is another name for Pre-load?

systemic vascular resistance (SVR)

Following a thyroidectomy, a patient complains of "a tingling feeling around my mouth." Which assessment should the nurse complete first?

test for presence of Chvostek's sign

Type 1 juvenile diabetes is

the #1 endocrine disorder in children

What does the A in MONA stand for?

the A in MONA stands for aspirin

The nurse is caring for a client having respiratory distress related to an anxiety attack. Recent ABG values are: pH=7.53, PaO2=72, PaCO2= 32, HCO3=28. Which conclusion about the client should the nurse make?

the client is probably hyperventilating

The nurse analyzes the results of a patient's ABG. Which finding requires immediate action?

the partial pressure of oxygen in arterial blood (PaO2) is 62

A pregnant patient with eclampsia is receiving IV magnesium sulfate. Which finding should the nurse report to the health care provider immediately?

the patellar and triceps reflexes are absent

device placed on radial cath site

tourniquet

>70% blockage in 3 or more coronary arteries

triple vessel disease

Lab value that is most indicative of a MI

troponin

Positive results on ICAs and GAD antibody test means

type 1 diabetes

NHL treatments are driven by

type and location of cancer...usually takes chemo and radiation

atorvastatin reduces the risk of recurrent myocardial infarction and has a common side effect of

unexplained muscle pain, cramping, or weakness

angina that is constant, even at rest

unstable

The nurse is making a home visit to a child who has a chronic disease. Which finding has the most implication for acid-base aspects of this patients care?

urine output is very small today

Diabetes diagnostics include

urine tests (checks for glucose, ketones, and microalbuminuria which is protein); serum glucose, glucose tolerance test, glycosylated hemoglobin (A1C), immunoassays (differentiate between type 1 and type 2)

What is the type of drug used to treat hypertensive emergency and includes Hydralazine, Sodium Nitroprusside, and Nitroglycerin?

vasodilators

ACE inhibitors are given after an MI to reduce what?

ventricular remodeling

Risk of death in diabetics is 2x that of a person

without diabetes

If Blood glucose is over 250 or your spilling ketones, it is recommended that you not exercise because

you're vascularly dry and may pass out


Related study sets

Articulations: Selected Articulations in Depth

View Set

Chapter 3- Colonies Take Root (Review Questions)

View Set

Principles of Meat Science: Deterioration, Preservation, and Storage (Lecture 10 notes)

View Set

algebra 2b - unit 5: more than one function

View Set

State Exam Simulator : 200 questions

View Set

Module 1 Lifeguard Training: Pectora

View Set

Bio 180 Practice Questions and Vocab

View Set